Convergence of improper integrals with parameters

Click For Summary
The discussion centers on the convergence of the integral F(t_1, t_2) = ∫_0^1 x^{t_1} ln^{t_2}(1/x) dx, with the conclusion that it converges for t in the range (-1, ∞)^2. Participants explore various convergence tests, including the limit approach and the relationship between functions f and g. A change of variable to u = ln(1/x) simplifies the analysis, leading to the conclusion that the integral converges based on the behavior of the exponential term e^{-(t_1+1)u}. The discussion also touches on the gamma function's role in convergence, particularly when t1 = 0, and raises questions about other potential values of t1 for t2 < -1. Understanding these convergence conditions is essential for analyzing improper integrals with parameters.
hamsterman
Messages
74
Reaction score
0
I'm having a lot of trouble with the subject. Here's one example I'd like explained.
F(t_1, t_2) = \int \limits_0^1 x^{t_1}\ln^{t_2}\frac{1}{x} dx
The book asks to find for what \vec{t} F converges. The answer is \vec{t}\in(-1; \infty)^2, but I don't see how to get that.

In general, what tools are there to test convergence? The only one I know is that (assuming that only f(b) is undefined) \int_a^b f converges if \lim \limits_{x \rightarrow b} \int_x^b f = 0.
Is it safe to assume that if g \sim f, x \rightarrow b and \int g converges then so does \int f?

My book shows some tests for uniform convergence, but that's not exactly the same thing, is it?
 
Physics news on Phys.org
use the change of variable u=ln(1/x), it becomes much clearer.
 
So that's \int f = \int \limits_0^{\infty}e^{-(t_1+1)u}u^{t_2} \mathrm{d} u then.
It seems clear that \lim \limits_{u \rightarrow 0} f = u^{t_2} and f converges at infinity when e^{-(t_1+1)u} does.
However, when t1 = 0, this integral is gamma function of t2+1. It also converges on negative non integers. Why did I not find that? Also, do there exist other values of t1 such that the integral converges for some t2<-1 ?
 
hamsterman said:
So that's \int f = \int \limits_0^{\infty}e^{-(t_1+1)u}u^{t_2} \mathrm{d} u then.
It seems clear that \lim \limits_{u \rightarrow 0} f = u^{t_2} and f converges at infinity when e^{-(t_1+1)u} does.
However, when t1 = 0, this integral is gamma function of t2+1. It also converges on negative non integers. Why did I not find that? Also, do there exist other values of t1 such that the integral converges for some t2<-1 ?

According to this
http://en.wikipedia.org/wiki/Gamma_function
The integral representation of Gamma function is convergent only if t2+1>0. The full Gamma function is obtained by analytic continuation.
 
Thanks, that's good to know.
 
Question: A clock's minute hand has length 4 and its hour hand has length 3. What is the distance between the tips at the moment when it is increasing most rapidly?(Putnam Exam Question) Answer: Making assumption that both the hands moves at constant angular velocities, the answer is ## \sqrt{7} .## But don't you think this assumption is somewhat doubtful and wrong?

Similar threads

  • · Replies 2 ·
Replies
2
Views
2K
  • · Replies 4 ·
Replies
4
Views
2K
  • · Replies 105 ·
4
Replies
105
Views
6K
  • · Replies 6 ·
Replies
6
Views
2K
  • · Replies 15 ·
Replies
15
Views
2K
  • · Replies 1 ·
Replies
1
Views
2K
  • · Replies 5 ·
Replies
5
Views
2K
Replies
2
Views
2K
  • · Replies 11 ·
Replies
11
Views
2K
  • · Replies 8 ·
Replies
8
Views
2K